Download as pptx, pdf, or txt
Download as pptx, pdf, or txt
You are on page 1of 15

Data Sufficiency

Data Sufficiency question will have the following structure:


Q. A question will be given.
Statement 1 – Some information will be given.
Statement 2 – Some information will be given.
On the basis of the statements given, four or five different options will be given. And one of
those options will be the most probable choice and hence the answer.
Tips to Solve Data Sufficiency Questions
The candidates are supposed to find out which of the given statements is or are sufficient to
solve the given question
This section tests the candidate’s efficiency to determine the information necessary
to solve the given problem.
Step1: Take Statement 1 and try to reach a conclusion.
Step 2: Take Statement 2 and try to reach a conclusion. Information given in statement 1 is
NOT used in this step.
Step 3: If we are not able to reach a conclusion in step 1 or step 2, take both statements
together and try to reach a conclusion

Give answer
(A) If the data in statement I alone are sufficient to answer the question, while the data in
statement II alone are not sufficient to answer the question
(B) If the data in statement II alone are sufficient to answer the question, while the data in
statement I alone are not sufficient to answer the question
(C) If the data either in statement I alone or in statement II alone are sufficient to answer the
question
(D) If the data given in both statements I and II together are not sufficient to answer the
question
(E) If the data in both statements I and II together are necessary to answer the question.
Question 1: Is x > 3?
Statement 1: x is the smallest prime.
Statement 2: x2=4
Answer
Using statement 1 alone: smallest prime is 2. Hence x = 2, we can conclude that x is NOT greater
than 3. Remember that getting No is also a conclusion.

Using statement 2 alone: √4 = +2 or -2. Both case x is NOT greater than 3.


As we can conclude in both step1 and step2 we don’t have to go for the step 3.

So either of the statement can give conclusion alone


DIRECTION: In each of the Questions below consists of a Question and two Statements
numbered I and II given below it. You have to decide whether the data provided in the
Statements are sufficient to answer the Question. Read both the Statements and
(A) If the data in statement I alone are sufficient to answer the Question, while the data in
statement II alone are not sufficient to answer the Question
(B) If the data in statement II alone are sufficient to answer the Question, while the data in
statement I alone are not sufficient to answer the Question
(C) If the data either in statement I alone or in statement II alone are sufficient to answer the
Question
(D) If the data given in both Statements I and II together are not sufficient to answer the
Question
(E) If the data in both Statements I and II together are necessary to answer the Question.

Question: What is the shortest distance between Devipur and Durgapur ?


Statements:
1.Durgapur is 20 kms away from Rampur.
2.Devipur is 15 kms away from Rampur.
DIRECTION: In each of the Questions below consists of a Question and two Statements
numbered I and II given below it. You have to decide whether the data provided in the
Statements are sufficient to answer the Question. Read both the Statements and
(A) If the data in statement I alone are sufficient to answer the Question, while the data in
statement II alone are not sufficient to answer the Question
(B) If the data in statement II alone are sufficient to answer the Question, while the data in
statement I alone are not sufficient to answer the Question
(C) If the data either in statement I alone or in statement II alone are sufficient to answer the
Question
(D) If the data given in both Statements I and II together are not sufficient to answer the
Question and
(E) If the data in both Statements I and II together are necessary to answer the Question.

Question: Among Monika, Anita, Sonal, Ratna and Tanvy, who came last for the programme ?
Statements:
1.Monika came after Anita but not after Tanvy.
2.Ratna came after Tanvy but not after Sonal.
DIRECTION: In each of the Questions below consists of a Question and two Statements
numbered I and II given below it. You have to decide whether the data provided in the
Statements are sufficient to answer the Question. Read both the Statements and
(A) If the data in statement I alone are sufficient to answer the Question, while the data in
statement II alone are not sufficient to answer the Question
(B) If the data in statement II alone are sufficient to answer the Question, while the data in
statement I alone are not sufficient to answer the Question
(C) If the data either in statement I alone or in statement II alone are sufficient to answer the
Question
(D) If the data given in both Statements I and II together are not sufficient to answer the
Question
(E) If the data in both Statements I and II together are necessary to answer the Question.

Question: On which date in August was Kapil born ?


Statements:
1.Kapil's mother remembers that Kapil was born before nineteenth but after fifteenth.
2.Kapil's brother remembers that Kapil was born before seventeenth but after twelfth.
DIRECTION: In each of the Questions below consists of a Question and two Statements
numbered I and II given below it. You have to decide whether the data provided in the
Statements are sufficient to answer the Question. Read both the Statements and
(A) If the data in statement I alone are sufficient to answer the Question, while the data in
statement II alone are not sufficient to answer the Question
(B) If the data in statement II alone are sufficient to answer the Question, while the data in
statement I alone are not sufficient to answer the Question
(C) If the data either in statement I alone or in statement II alone are sufficient to answer the
Question
(D) If the data given in both Statements I and II together are not sufficient to answer the
Question
(E) If the data in both Statements I and II together are necessary to answer the Question.

Question: Can Ritesh retire from office X in January 2006, with full pension benefits ?
Statements:
1.Ritesh will complete 30 years of service in office X in April 2000 and desires to retire.
2.As per office X rules, an employee has to complete minimum 30 years of service and attain age
of 60. Ritesh has 3 years to complete age of 60
DIRECTION: In each of the Questions below consists of a Question and two Statements
numbered I and II given below it. You have to decide whether the data provided in the
Statements are sufficient to answer the Question. Read both the Statements and
(A) If the data in statement I alone are sufficient to answer the Question, while the data in
statement II alone are not sufficient to answer the Question
(B) If the data in statement II alone are sufficient to answer the Question, while the data in
statement I alone are not sufficient to answer the Question
(C) If the data either in statement I alone or in statement II alone are sufficient to answer the
Question
(D) If the data given in both Statements I and II together are not sufficient to answer the
Question
(E) If the data in both Statements I and II together are necessary to answer the Question.

Question: Who is C's partner in a game of cards involving four players A, B, C and D ?
Statements:
1.D is sitting opposite to A.
2.B is sitting right of A and left of D.
DIRECTION: In each of the Questions below consists of a Question and two Statements
numbered I and II given below it. You have to decide whether the data provided in the
Statements are sufficient to answer the Question. Read both the Statements and
(A) If the data in statement I alone are sufficient to answer the Question, while the data in
statement II alone are not sufficient to answer the Question
(B) If the data in statement II alone are sufficient to answer the Question, while the data in
statement I alone are not sufficient to answer the Question
(C) If the data either in statement I alone or in statement II alone are sufficient to answer the
Question
(D) If the data given in both Statements I and II together are not sufficient to answer the
Question
(E) If the data in both Statements I and II together are necessary to answer the Question.

Question: What will be the total weight of 10 poles, each of the same weight ?
Statements:
1.One-fourth of the weight of each pole is 5 kg.
2.The total weight of three poles is 20 kilograms more than the total weight of two poles.
Question 2: Is A taller than B?
Statement 1: A is tallest in his class
Statement 2: A and B are class mates

Question 3. How many people are there in the plain?


Statement I: 25% passengers are women and 35% are children.
Statement II: There are 24 men in the plain
Q4. How is M related to N?
Statements:
I. B is the daughter of M and sister of Q.
II. N is the son of K who is B's grandfather.

Q5. How is R related to M?


Statements:
I. M's brother is husband of P.
II. P is mother of R's sister.
Q6. Is x divisible by 9?
Statements:
I. Sum of digits of x is divisible by 3
II. Sum of digits of x is divisible by 9

Q7. Is x divisible by 28?


Statements:
I. x is divisible by 20
II. x is divisible by 84
Q8. What is the difference between monthly income of Ram and Chaaru?
Statements
I. Ram earns Rs 6000 less than Shaam
II. Chaaru earns Rs 6000 more than Shaam.

Q9. What is the code for 'is' in the code language?


Statements:
I. In the code language, 'shi tu ke' means 'pen is blue'.
II. In the same code language, 'ke si re' means 'this is wonderful'.
Q10. Among A, B, C, D, E and F, who is the heaviest?
Statements:
I. A and D are heavier than B, E and F but none of them is the heaviest.
II. A is heavier than D but lighter than C.

Q11. What is the code for 'or' in the code language?


Statements:
I. 'nik sa te' means 'right or wrong', 'ro da nik' means 'he is right' and 'fe te ro' means 'that is
wrong'.
II. 'pa nik la' means 'that right man', 'sa ne pa' means 'this or that' and 'ne ka re' means 'tell this
there'.

You might also like